Đến nội dung

phamhuy1801

phamhuy1801

Đăng ký: 25-07-2015
Offline Đăng nhập: 25-07-2019 - 13:52
***--

#708706 [TOPIC] HÌNH HỌC ÔN THI VÀO THPT CHUYÊN 2018-2019

Gửi bởi phamhuy1801 trong 18-05-2018 - 20:08

Bài $58$: Cho $\triangle ABC$ vuông tại $A$, đường cao $AH$. $HD, HE$ lần lượt là phân giác góc $BHA$ và $CHA$ ($D,E$ thuộc $AB, AC$). $I$ là trung điểm $DE$. $BI$ cắt $DH, CD$ lần lượt tại $M,P$; $CI$ cắt $EH$, $BE$ lần lượt tại $N,Q.$ $BE$ cắt $CD$ tại $K.$ Chứng minh:

a, Tứ giác $APKQ$ nội tiếp.
b*, $MN//DE$ và $MN$ cắt $AH$ tại $K$.
32588593_378688822634545_155765427741183

 

 

Bài $58a$.
Dễ chứng minh được $\triangle ADE$ vuông cân tại A, do đó $2AE.AD=DE^2$
Theo định lí đường phân giác trong và hệ thức lượng:
$\frac{BD}{DA}=\frac{BH}{HA}=\frac{HA}{HC}=\frac{AE}{EC}$
Suy ra $BD.EC=AD.AE=\frac{DE^2}{2}=DE.DI$
$\leftrightarrow  \frac{BD}{DI}=\frac{DE}{EC}$
$\rightarrow \triangle BDI \sim \triangle DEC$
$\rightarrow  \widehat{DIP}=\widehat{ECP}$
Do đó tứ giác $PIEC$ nội tiếp.
$\leftrightarrow  DP.DC=DE.DI=DA^2$
$\leftrightarrow AP \perp DC$
Tương tự có $AQ \perp BE$.
Suy ra tứ giác $APKQ$ nội tiếp.
 
 



#708665 $\frac{a^2}{2a^2+bc}+\frac{b^2}...

Gửi bởi phamhuy1801 trong 18-05-2018 - 01:37

Cho các số dương a, b, c thì ta có bất đẳng thức:

 $\frac{a^2}{2a^2+bc}+\frac{b^2}{2b^2+ca}+\frac{c^2}{2c^2+ab}\leq 1$

 

p/s: Mọi người ủng hộ các bài toán có ứng dụng của BĐT này dùm em. Em cảm ơn 

 

C2: Sử dụng đánh giá $\frac{a^2}{2a^2+bc} \le \frac{a^2b^2+c^2a^2}{2(a^2b^2+b^2c^2+c^2a^2)}$.

Thiết lập các bất đẳng thức tương tự rồi cộng lại thu được đpcm.

 

C3: Đặt $(\frac{bc}{a^2}; \frac{ca}{b^2}; \frac{ab}{c^2}) \rightarrow (x;y;z)$

Khi đó ta có $xyz=1$ và quy về chứng minh:

$\frac{1}{x+2}+\frac{1}{y+2}+\frac{1}{z+2} \le 1$
Chứng minh đã có ở đây.
 
Trong cuốn "Sử dụng phương pháp C-S để chứng minh BĐT" của anh Cẩn có khá nhiều bài toán ứng dụng BĐT này  :D .



#708659 [TOPIC] HÌNH HỌC ÔN THI VÀO THPT CHUYÊN 2018-2019

Gửi bởi phamhuy1801 trong 17-05-2018 - 23:50

 

Bài $43$: Cho $\triangle ABC$ nhọn, đường cao $BD, CE$ cắt nhau tại $H$. Đường tròn đường kính $AB$ cắt $CE$ tại $G$, đường tròn đường kính $AC$ cắt $BD$ tại $F$. $CF$ cắt $BG$ tại $I$, $DG$ cắt $EF$ tại $K$.

a, Chứng minh $3$ điểm A,I,K cùng nằm trên đường trung trực của FG .

b, Giả sử $EF$ cắt $BG$ tại $M$, $DG$ cắt $CF$ tại $N$. Chứng minh $HI, EN, DM$ đồng quy.

 

32678503_378178532685574_842490035768708

 

Lời giải phần $b$ (có dùng định lí $Menelaus$ và định lí $sin$ trong tam giác)

Gọi $EN$ cắt $HI$ tại $P$. Ta chứng minh $M, P, D$ thẳng hàng. 

$\widehat{FMI}=\widehat{IGN}=\widehat{ABC}$, chứng minh được $\triangle FMI =\triangle GNI (g.c.g) \rightarrow MI=NI; \widehat{FMI}=\widehat{ING}$

Ta có:

$\frac{BM}{BE}=\frac{sin \widehat{BEF} }{sin \widehat{BME}}=\frac{sin \widehat{ACF} }{sin \widehat{FMI}}$
$\frac{CD}{CN}=\frac{sin \widehat{CND} }{sin \widehat{CDG}}=\frac{sin \widehat{ING} }{sin \widehat{ABG}}$
Nên:
$\frac{BM}{BE}.\frac{CD}{CN}=\frac{sin \widehat{ACF} }{sin \widehat{FMI}}.\frac{sin \widehat{ING} }{sin \widehat{ABG}}=\frac{sin \widehat{ACF} }{sin \widehat{ABG}}$
$\Leftrightarrow \frac{BM}{BE}.\frac{CD}{CN}=\dfrac{(\dfrac{AF}{AC})}{(\dfrac{AG}{AB})}=\frac{AB}{AC}$
$\Leftrightarrow \frac{BM}{CN}=\frac{AB}{AC}.\frac{BE}{CD}$
$\Leftrightarrow \frac{BM}{CN}=\frac{HE}{EC}.\frac{DB}{HD}$
$\Leftrightarrow \frac{HD}{DB}.\frac{BM}{MI}=\frac{HE}{EC}.\frac{CN}{NI}$ (sử dụng $MI=NI$) 
$\Leftrightarrow \frac{IP}{PH}.\frac{HD}{DB}.\frac{BM}{MI}=1$ (theo định lí $Menelaus$ có $\frac{HE}{EC}.\frac{CN}{NI}.\frac{PI}{PH}=1$)

$\Leftrightarrow $ $M,D,P$ thẳng hàng. Bài toán được chứng minh.




#708564 [TOPIC] HÌNH HỌC ÔN THI VÀO THPT CHUYÊN 2018-2019

Gửi bởi phamhuy1801 trong 16-05-2018 - 21:27

Bài $57$: Cho $\triangle ABC$ có $I$ là tâm đường tròn nội tiếp. Đường trung trực đoạn $IC$ cắt $AI, BI, AC, BC$ lần lượt tại $D,E,H,F$. Gọi $T$ là tâm đường tròn ngoại tiếp $\triangle AIH$.
a, Chứng minh $(T)$ đi qua $E$ và $5$ điểm $A,B,C,D,E$ cùng thuộc một đường tròn.
b, Chứng minh $IT \perp BD$.
32650032_378688749301219_208224515206309
Bài $58$: Cho $\triangle ABC$ vuông tại $A$, đường cao $AH$. $HD, HE$ lần lượt là phân giác góc $BHA$ và $CHA$ ($D,E$ thuộc $AB, AC$). $I$ là trung điểm $DE$. $BI$ cắt $DH, CD$ lần lượt tại $M,P$; $CI$ cắt $EH$, $BE$ lần lượt tại $N,Q.$ $BE$ cắt $CD$ tại $K.$ Chứng minh:
a, Tứ giác $APKQ$ nội tiếp.
b*, $MN//DE$ và $MN$ cắt $AH$ tại $K$.
32588593_378688822634545_155765427741183



#708556 [TOPIC] ÔN THI BẤT ĐẲNG THỨC $\boxed{\text{THPT CHUYÊN}}$...

Gửi bởi phamhuy1801 trong 16-05-2018 - 20:32

Khuấy đảo topic lại nào

$\boxed{\text{Bài 125}}$ Cho a,b,c là các số thực dương

Chứng minh $$\frac{2a}{b+c}+\frac{2b}{a+c}+\frac{2c}{a+b} \geq 3+\frac{(a-b)^2+(b-c)^2+(c-a)^2}{(a+b+c)^2}$$

 

Ta đã biết:

$\frac{2a}{b+c}+\frac{2b}{c+a}+\frac{2c}{a+b}-3=\frac{(a-b)^2}{(c+a)(c+b)}+\frac{(b-c)^2}{(a+b)(a+c)}+\frac{(c-a)^2}{(b+c)(b+a)}$
Nên bất đẳng thức ban đầu tương đương với:
$(a-b)^2[\frac{1}{(c+a)(c+b)}-\frac{1}{(a+b+c)^2}]+(b-c)^2[\frac{1}{(a+b)(a+c)}-\frac{1}{(a+b+c)^2}]+(c-a)^2[\frac{1}{(b+a)(b+c)}-\frac{1}{(a+b+c)^2}] \ge 0$
$\Leftrightarrow (a-b)^2\frac{(a^2+b^2+ab+bc+ca)}{(c+a)(c+b)(a+b+c)^2}+(b-c)^2\frac{(b^2+c^2+ab+bc+ca)}{(a+b)(a+c)(a+b+c)^2}+(c-a)^2\frac{(a^2+c^2+ab+bc+ca)}{(b+a)(b+c)(a+b+c)^2} \ge 0$ (đúng với $a,b,c$ dương)



#708446 [TOPIC] HÌNH HỌC ÔN THI VÀO THPT CHUYÊN 2018-2019

Gửi bởi phamhuy1801 trong 15-05-2018 - 16:10

Một số bài toán trong đây phức tạp hơn mình nghĩ  :D Vẫn còn nhé.

 

Bài $42$: (4 bài toán đơn giản, độc lập)

$a,$ Cho $\triangle ABC$ nội tiếp $(O)$, phân giác góc $BAC$ cắt $(O)$ tại $D$. Gọi $N, K$ lần lượt là chân các đường vuông góc kẻ từ $O, D$ xuống $AC$. Chứng minh $AB=2NK$.

32646817_378178492685578_586927716882291

$b,$ Cho $(O)$ và dây cung $BC$ cố định, lấy điểm $A$ trên cung lớn $BC$. Kẻ $BD \perp AC, CE \perp AB$. $BD$ cắt đường thẳng kẻ từ $A$ vuông góc với $AB$ tại $I$, $ED$ cắt đường thẳng kẻ từ $A$ song song với $BC$ tại $M$. Chứng minh $MI$ đi qua tâm một đường tròn có đường kính cố định.

32580672_378178522685575_699663071593680

$c,$ Cho $\triangle ABC$ nhọn $(AB < AC)$, đường cao $BD, CE$ cắt nhau tại $H$. $M,N$ lần lượt là trung điểm $BD,CE$. Gọi $I$ là trực tâm $\triangle ADE$. Chứng minh $MN \perp HI$

$d,$ Cho $\triangle ABC$ nội tiếp $(O)$. $H,K$ lần lượt là trung điểm $AB, AC$. $AO$ cắt $HK$ tại $I$. $CI$ cắt $KO$ tại $E$, $BI$ cắt $OH$ tại $F$. Chứng minh $EF//KH$.
 
Bài $43$: Cho $\triangle ABC$ nhọn, đường cao $BD, CE$ cắt nhau tại $H$. Đường tròn đường kính $AB$ cắt $CE$ tại $G$, đường tròn đường kính $AC$ cắt $BD$ tại $F$. $CF$ cắt $BG$ tại $I$, $DG$ cắt $EF$ tại $K$.
a, Chứng minh $3$ điểm A,I,K cùng nằm trên đường trung trực của FG .

b, Giả sử $EF$ cắt $BG$ tại $M$, $DG$ cắt $CF$ tại $N$. Chứng minh $HI, EN, DM$ đồng quy.

 

32678503_378178532685574_842490035768708




#708380 [TOPIC] HÌNH HỌC ÔN THI VÀO THPT CHUYÊN 2018-2019

Gửi bởi phamhuy1801 trong 14-05-2018 - 20:53

Bài $35$: Cho $\triangle ABC$ nhọn. Đường tròn $(O; \dfrac{BC}{2} )$ cắt $AB, AC$ thứ tự tại $E,D$. Các tiếp tuyến kẻ từ $E,D$ của $(O)$ cắt nhau tại $I$ và lần lượt cắt tiếp tuyến kẻ từ $B, C$ của $(O)$ tại $F,G$. $FG$ cắt $AI$ tại $H$ và cắt $(O)$ tại $M,N$. Chứng minh $H$ là trực tâm của $\triangle ABC $ và $MA, NA$ lần lượt các tiếp tuyến của $(O)$.
 
32416434_377856542717773_785201938761043
Bài $36$: Từ điểm $M$ ngoài $(O)$, vẽ tiếp tuyến $MA, MB$. Đường tròn $(I)$ nội tiếp $\triangle ABM$ tiếp xúc với $AB, AM$ lần lượt tại $H,G$ ; $(I)$ cắt $(O)$ tại $P,R$ ($P$ nằm trên nửa mặt phẳng bờ $OM$ chứa $A$), $AR$ cắt $(I)$ tại $Q$. Chứng minh $PR, AI, HG$ đồng quy và $\triangle APQ$ cân.
 
32545777_377856576051103_620914316652052
 



#707879 Chứng minh $DE//CF$

Gửi bởi phamhuy1801 trong 07-05-2018 - 23:10

Cho $\triangle ABC$ cân tại $A$ có $\widehat{A}=20^{\circ}$. Trên $AB$ lấy hai điểm $D$ và $F$, trên $AC$ lấy điểm $E$ sao cho $AD=BF=BC$, $EA=EB$. Chứng minh $DE//CF$. 

Gọi $I$ là điểm chung thứ $2$ của đường tròn ngoại tiếp $\triangle EDF$ và $\triangle FBC$. Chứng minh $FI \perp AC$.




#707352 [TOPIC] ÔN THI BẤT ĐẲNG THỨC $\boxed{\text{THPT CHUYÊN}}$...

Gửi bởi phamhuy1801 trong 30-04-2018 - 06:57

Bài 103: Cho a,b,c không âm

Cmr: $a^2+b^2+c^2+3\sqrt[3]{(abc)^2}\geq 2(ab+bc+ac)$

P/s: TT2 là gì vậy. Có phải là Toán Học Tuổi Trẻ ko ?

 

$a+b+c \ge 3 \sqrt[3]{abc} \Leftrightarrow 3\sqrt[3]{(abc)^2} \ge \frac{9abc}{a+b+c}$

Do đó chỉ cần chỉ ra $a^2+b^2+c^2+\frac{9abc}{a+b+c} \ge 2(ab+bc+ca)$, đây là một dạng tương đương của Schur bậc $3$.

 

Bài $111$: Cho các số dương $a,b,c$ thỏa mãn $abc=1$. Chứng minh:

$\frac{1}{a+1}+\frac{1}{b+1}+\frac{1}{c+1}-\frac{1}{2} \ge \frac{1}{a+2}+\frac{1}{b+2}+\frac{1}{c+2} \ge \frac{a}{a^2+a+1}+\frac{b}{b^2+b+1}+\frac{c}{c^2+c+1}$




#707191 $\frac{1}{a+2}+\frac{1}{b+2...

Gửi bởi phamhuy1801 trong 28-04-2018 - 17:07

Cho các số dương $a, b,c$ thỏa mãn $abc=1$. Chứng minh rằng: 

$\frac{1}{a+2}+\frac{1}{b+2}+\frac{1}{c+2}\leq 1$

- Sáng tác -

 

Quy đồng nhân tung ta thu được $ab+bc+ca \ge 3$, hiển nhiên đúng với điều kiện đề bài.

 

Với điều kiện $a,b,c$ dương, $abc=1$, mình cũng tìm ra chuỗi bất đẳng thức khá thú vị :D

$\frac{1}{a+1}+\frac{1}{b+1}+\frac{1}{c+1}-\frac{1}{2} \ge \frac{1}{a+2}+\frac{1}{b+2}+\frac{1}{c+2} \ge \frac{a}{a^2+a+1}+\frac{b}{b^2+b+1}+\frac{c}{c^2+c+1}$




#706725 [TOPIC] ÔN THI BẤT ĐẲNG THỨC $\boxed{\text{THPT CHUYÊN}}$...

Gửi bởi phamhuy1801 trong 23-04-2018 - 12:54

Bài $65$: Cho các số dương $x,y,z$. Chứng minh: 

                                               $(x+\frac{yz}{x})(y+\frac{zx}{y})(z+\frac{xy}{z}) \ge 4\sqrt[3]{(x^3+y^3)(y^3+z^3)(z^3+x^3)}$

 

Mình xin đưa ra lời giải cho bài toán của mình:

$(x+\frac{yz}{x})(y+\frac{zx}{y})(z+\frac{xy}{z}) \ge 4\sqrt[3]{(x^3+y^3)(y^3+z^3)(z^3+x^3)}$
Chia cả hai vế cho $xyz$ ta được:
$\Leftrightarrow (\frac{x}{y}+\frac{z}{x})(\frac{y}{z}+\frac{x}{y})(\frac{z}{x}+\frac{y}{z}) \ge 4\sqrt[3]{(\frac{x^3}{y^3}+1)(\frac{y^3}{z^3}+1)(\frac{z^3}{x^3}+1)} $
 
Đặt $(\frac{x}{y}; \frac{y}{z}; \frac{z}{x}) \rightarrow (a,b,c)$ thì $abc=1$ và ta sẽ chứng minh:
$\Leftrightarrow (a+b)(b+c)(c+a) \ge 4 \sqrt[3]{(a^3+1)(b^3+1)(c^3+1)}$
$\Leftrightarrow [(a+b)(b+c)(c+a)]^3 \ge 64(a^3+abc)(b^3+abc)(c^3+abc) $
$\Leftrightarrow \sqrt{[(a+b)(b+c)(c+a)]^3} \ge 8\sqrt{abc(a^2+bc)(b^2+ca)(c^2+ab)}$             $($  :D $)$
 
Xét bất đẳng thức: 
$(a+b)(b+c) = a^2+bc + b(c+a) \ge 2\sqrt{b(c+a)(a^2+bc)} $
Thiết lập các bất đẳng thức tương tự rồi nhân theo vế, ta thu được $[(a+b)(b+c)(c+a)]^2 \ge 8\sqrt{a(b+c)(a^2+bc).b(c+a)(b^2+ca).c(a+b)(c^2+ab)}$, đây chính là bất đẳng thức $($  :D $)$, bài toán được chứng minh.



#706675 [TOPIC] ÔN THI BẤT ĐẲNG THỨC $\boxed{\text{THPT CHUYÊN}}$...

Gửi bởi phamhuy1801 trong 22-04-2018 - 20:26

Bài 74: 

Cho a, b, c là 3 số thực không âm. Chứng minh:

$\frac{a^4}{a^2+ab+b^2}+\frac{b^4}{b^2+bc+c^2}+\frac{c^4}{c^2+ac+c^2} \ge \frac{a^3+b^3+c^3}{a+b+c} $

 

$\frac{a^4}{a^2+ab+b^2}+\frac{b^4}{b^2+bc+c^2}+\frac{c^4}{c^2+ac+c^2} \ge \frac{a^3+b^3+c^3}{a+b+c} $
$\Leftrightarrow \frac{a^4}{a^2+ab+b^2}-a(a-b)+\frac{b^4}{b^2+bc+c^2}-b(b-c)+\frac{c^4}{c^2+ac+c^2}-c(c-a) \ge \frac{a^3+b^3+c^3-(a+b+c)(a^2+b^2+c^2-ab-bc-ca)}{a+b+c} $
$\Leftrightarrow \frac{ab^3}{a^2+ab+b^2}+\frac{bc^3}{b^2+bc+c^2}+\frac{ca^3}{c^2+ac+c^2} \ge \frac{3abc}{a+b+c}$
 
Bất đẳng thức trên đúng vì:
 
$\frac{ab^3}{a^2+ab+b^2}+\frac{bc^3}{b^2+bc+c^2}+\frac{ca^3}{c^2+ac+c^2} = \frac{ab^3c}{c(a^2+ab+b^2)}+\frac{abc^3}{a(b^2+bc+c^2)}+\frac{bca^3}{b(c^2+ac+c^2)} \ge \frac{(\sum a\sqrt{abc})^2}{\sum a(b^2+bc+c^2)} = \frac{abc(a+b+c)^2}{(a+b+c)(ab+bc+ca)} \ge \frac{abc.3(ab+bc+ca)}{(a+b+c)(ab+bc+ca)} = \frac{3abc}{a+b+c} $



#706608 [TOPIC] ÔN THI BẤT ĐẲNG THỨC $\boxed{\text{THPT CHUYÊN}}$...

Gửi bởi phamhuy1801 trong 21-04-2018 - 21:18

Bài $64$: (Tìm giá trị nhỏ nhất)

$ P=\sum \frac{x^3+y^3}{xy+9} = \sum x^3\left (\frac{1}{xy+9}+\frac{1}{xz+9} \right ) $

$ \geq \sum \frac{4x^3}{x(y+z)+18} = \sum \frac{4x^3}{x(9-x)+18} = \sum \frac{4x^3}{-x^2+9x+18} $

Có thể chỉ ra $\frac{4x^3}{-x^2+9x+18} \geq \frac{11x-21}{4} \Leftrightarrow (x-3)^2(9x+14) \geq 0$ bằng phương pháp cân bằng hệ số hoặc phương pháp tiếp tuyến.

Cộng lại các bất đẳng thức tương tự, $P \geq 9$

 

Đóng góp thêm 1 bài toán (made by me từ hồi lớp 9 :3)

 

Bài $65$: Cho các số dương $x,y,z$. Chứng minh: 

                                               $(x+\frac{yz}{x})(y+\frac{zx}{y})(z+\frac{xy}{z}) \ge 4\sqrt[3]{(x^3+y^3)(y^3+z^3)(z^3+x^3)}$




#705023 ĐỀ THI HỌC SINH GIỎI TỈNH QUẢNG BÌNH NĂM 2017-2018

Gửi bởi phamhuy1801 trong 06-04-2018 - 00:26

Có thể làm mạnh bài toán bằng cách chứng minh giá trị lớn nhất của vế trái bằng $\sqrt[3]{6}$, nghiễm nhiên nhỏ hơn 2.

Đặt $P=\sum \sqrt[3]{\frac{2a}{4a+4b+c}}$
$3\sqrt[3]{\frac{9}{2}}.P= \sum 3.\sqrt[3]{\frac{9a}{4a+4b+c}.1.1} \le \sum \frac{9a}{4a+4b+c}+6 $  
 
Ta sẽ chứng minh $\sum \frac{9a}{4a+4b+c} \le 3 \Leftrightarrow \sum \frac{3a(4a+4b+4c)}{4a+4b+c} \le 4(a+b+c) \Leftrightarrow \sum \frac{9ca}{4a+4b+c} \le a+b+c$
Điều này đúng vì $\sum \frac{9ca}{4a+4b+c} \le \sum (\frac{2ca}{2a+b}+ \frac{ca}{2b+c}) = a+b+c$
 
Vậy $3\sqrt[3]{\frac{9}{2}}.P \le 9$ hay $P \le \sqrt[3]{6} < 2 $. 



#704108 \[\frac{a}{b+c}+\frac{b}{a+...

Gửi bởi phamhuy1801 trong 22-03-2018 - 21:49

Bất đẳng thức tương đương với

$\sum \dfrac{2a}{b+c}-3 \geq  \frac{6(a^2+b^2+c^2)}{(a+b+c)^2}-2 $
$\leftrightarrow \sum \frac{(a-b)^2}{(a+c)(b+c)} \ge \frac{\sum 2(a-b)^2}{(a+b+c)^2} $
$\leftrightarrow \sum (a-b)^2\frac{a^2+b^2-c^2}{(a+b+c)^2(a+c)(b+c)} \ge 0$ 
$(S_a; S_b; S_c) \rightarrow (\frac{c^2+b^2-a^2}{(a+b+c)^2(a+c)(a+b)};\frac{c^2+a^2-b^2}{(a+b+c)^2(b+c)(b+a)} ;\frac{a^2+b^2-c^2}{(a+b+c)^2(a+c)(b+c)} )$
Giả sử $c \ge b \ge a$ thì $S_a \ge 0$, ta sẽ chỉ ra $b^2S_b + c^2S_c \ge 0 \leftrightarrow  c^2(a^2+b^2-c^2)(a+b) + b^2(c^2+a^2-b^2)(c+a) \ge 0$, chứng minh nhờ $c \ge b \ge a$.